Is a Bounded Set in R Countable if it Can be Covered by an Epsilon Cover?

  • Thread starter Thread starter sayan2009
  • Start date Start date
  • Tags Tags
    Bounded Set
Click For Summary
SUMMARY

The discussion centers on proving that a bounded subset A of the real numbers R is countable if it can be covered by an epsilon cover consisting of countable elements. The approach involves utilizing the properties of totally bounded sets and compactness of closed intervals. Specifically, the solution suggests covering A with open intervals that intersect A countably and leveraging the compactness of a closed interval containing A to establish the countability of A.

PREREQUISITES
  • Understanding of bounded sets in real analysis
  • Familiarity with the concept of epsilon covers
  • Knowledge of compactness in topology
  • Basic principles of countability in set theory
NEXT STEPS
  • Study the properties of totally bounded sets in real analysis
  • Learn about epsilon-delta definitions and their applications
  • Research compactness in metric spaces and its implications
  • Explore countability and its various forms in set theory
USEFUL FOR

Mathematicians, students studying real analysis, and anyone interested in set theory and topology will benefit from this discussion.

sayan2009
Messages
14
Reaction score
0

Homework Statement



given a set A(subset of R(reals)) is bounded.and for all x belongs to R there exists epsilon(eps) such that {(x-eps,x+eps) intersection A} is countable..to prove A is countable

Homework Equations





The Attempt at a Solution

...bdd set in R is totally bounded...but iam not finding the way how to cover A by epsilon cover(has at most countable elements)
 
Physics news on Phys.org
Take a closed interval C containing A and cover it with open intervals having a countable intersection with A. Now use the compactness of the closed interval C.
 
oooooooooooo gr888888888...thanks
 
Question: A clock's minute hand has length 4 and its hour hand has length 3. What is the distance between the tips at the moment when it is increasing most rapidly?(Putnam Exam Question) Answer: Making assumption that both the hands moves at constant angular velocities, the answer is ## \sqrt{7} .## But don't you think this assumption is somewhat doubtful and wrong?

Similar threads

Replies
1
Views
2K
Replies
16
Views
2K
Replies
4
Views
2K
  • · Replies 1 ·
Replies
1
Views
2K
Replies
1
Views
3K
  • · Replies 3 ·
Replies
3
Views
3K
  • · Replies 16 ·
Replies
16
Views
2K
  • · Replies 1 ·
Replies
1
Views
2K
  • · Replies 25 ·
Replies
25
Views
3K
  • · Replies 18 ·
Replies
18
Views
3K